A curious equality of integrals involving the prime counting function?How many primes does this sequence...

Why is it that Bernie Sanders is always called a "socialist"?

How would an AI self awareness kill switch work?

Why do we have to make "peinlich" start with a capital letter and also end with -s in this sentence?

Changing the laptop's CPU. Should I reinstall Linux?

Is this ordinary workplace experiences for a job in Software Engineering?

How to politely refuse in-office gym instructor for steroids and protein

After checking in online, how do I know whether I need to go show my passport at airport check-in?

How to make ice magic work from a scientific point of view?

What is a good reason for every spaceship to carry a weapon on board?

Limits of a density function

Can I announce prefix 161.117.25.0/24 even though I don't have all of /24 IPs

Why didn't Tom Riddle take the presence of Fawkes and the Sorting Hat as more of a threat?

Is there a verb that means to inject with poison?

How does one write from a minority culture? A question on cultural references

Why was Lupin comfortable with saying Voldemort's name?

Why did Luke use his left hand to shoot?

Is "the fire consumed everything on its way" correct?

Non-Cancer terminal illness that can affect young (age 10-13) girls?

Which communication protocol is used in AdLib sound card?

What happens when I Twin Life Transference?

Why are all my replica super soldiers young adults or old teenagers?

Is there a defined priority for pattern matching?

What is the difference between rolling more dice versus fewer dice?

Why zero tolerance on nudity in space?



A curious equality of integrals involving the prime counting function?


How many primes does this sequence find?Tight bounds on the prime counting functionDirichlet prime counting function?closed form for integrals involving error functiona practical prime counting functionPrime counting functionRestricted equality involving prime numbersPrime counting function formulasProof for a prime number formula involving the prime counting functionProperty of Prime Counting FunctionApproximating the prime counting function













9












$begingroup$


This post discusses the integral,
$$I(k)=int_0^kpi(x)pi(k-x)dx$$



where $pi(x)$ is the prime-counting function. For example,
$$I(13)=int_0^{13}pi(x)pi(13-x)dx = 73$$



Using WolframAlpha, the first 50 values for $k=1,2,3,dots$ are,



$$I(k) = 0, 0, 0, 0, 1, 4, 8, 14, 22, 32, 45, 58, 73, 90, 110, 132, 158, 184, 214, 246, 282, 320, 363, 406, 455, 506, 562, 618, 678, 738, 804, 872, 944, 1018, 1099, 1180, 1269, 1358, 1450, 1544, 1644, 1744, 1852, 1962, 2078, 2196, 2321, 2446, 2581, 2718,dots$$



While trying to find if the above sequence obeyed a pattern, I noticed a rather unexpected relationship:






Q: For all $n>0$, is it true,
$$I(6n+4) - 2,I(6n+5) + I(6n+6) overset{color{red}?}= 0$$




Example, for $n=1,2$, then
$$I(10)-2I(11)+I(12)=32-2*45+58 = 0$$
$$I(16)-2I(17)+I(18)=132-2*158+184= 0$$
and so on.










share|cite|improve this question











$endgroup$












  • $begingroup$
    Note your proposed equation doesn't hold for $n = 0$ as $I(4) = 0$, $I(5) = 1$ and $I(6) = 4$.
    $endgroup$
    – John Omielan
    2 hours ago










  • $begingroup$
    @JohnOmielan: A typo. I meant all $n>0$. I will correct it.
    $endgroup$
    – Tito Piezas III
    1 hour ago










  • $begingroup$
    I have checked to confirm what you're asking is true for $n$ up to $18$. However, I have my doubts it'll always work, partially because it doesn't work for $n = 0$. Also, a similar type condition is that $I(6n) - 2I(6n + 1) + I(6n + 2) = 2$, which holds for $1 le n le 5$, but at $n = 6$, the LHS becomes $0$ instead. If I get a chance, I will investigate your equation to see if I can figure out why it's true for at least the first $18$ values and, more importantly, will it always stay true. Regardless, though, it's an excellent observation you've made, even if it doesn't always hold.
    $endgroup$
    – John Omielan
    1 hour ago








  • 1




    $begingroup$
    I checked your result up to $n=533$ (for $n geq 534$, I have problems. Would you be interested by a huge table of $I(k)$ (I was able to generate it up to $k=540$). This is a very interesting problem.
    $endgroup$
    – Claude Leibovici
    59 mins ago












  • $begingroup$
    @ClaudeLeibovici: Thanks for checking, Claude! However, that table would be too huge for MSE. :)
    $endgroup$
    – Tito Piezas III
    56 mins ago
















9












$begingroup$


This post discusses the integral,
$$I(k)=int_0^kpi(x)pi(k-x)dx$$



where $pi(x)$ is the prime-counting function. For example,
$$I(13)=int_0^{13}pi(x)pi(13-x)dx = 73$$



Using WolframAlpha, the first 50 values for $k=1,2,3,dots$ are,



$$I(k) = 0, 0, 0, 0, 1, 4, 8, 14, 22, 32, 45, 58, 73, 90, 110, 132, 158, 184, 214, 246, 282, 320, 363, 406, 455, 506, 562, 618, 678, 738, 804, 872, 944, 1018, 1099, 1180, 1269, 1358, 1450, 1544, 1644, 1744, 1852, 1962, 2078, 2196, 2321, 2446, 2581, 2718,dots$$



While trying to find if the above sequence obeyed a pattern, I noticed a rather unexpected relationship:






Q: For all $n>0$, is it true,
$$I(6n+4) - 2,I(6n+5) + I(6n+6) overset{color{red}?}= 0$$




Example, for $n=1,2$, then
$$I(10)-2I(11)+I(12)=32-2*45+58 = 0$$
$$I(16)-2I(17)+I(18)=132-2*158+184= 0$$
and so on.










share|cite|improve this question











$endgroup$












  • $begingroup$
    Note your proposed equation doesn't hold for $n = 0$ as $I(4) = 0$, $I(5) = 1$ and $I(6) = 4$.
    $endgroup$
    – John Omielan
    2 hours ago










  • $begingroup$
    @JohnOmielan: A typo. I meant all $n>0$. I will correct it.
    $endgroup$
    – Tito Piezas III
    1 hour ago










  • $begingroup$
    I have checked to confirm what you're asking is true for $n$ up to $18$. However, I have my doubts it'll always work, partially because it doesn't work for $n = 0$. Also, a similar type condition is that $I(6n) - 2I(6n + 1) + I(6n + 2) = 2$, which holds for $1 le n le 5$, but at $n = 6$, the LHS becomes $0$ instead. If I get a chance, I will investigate your equation to see if I can figure out why it's true for at least the first $18$ values and, more importantly, will it always stay true. Regardless, though, it's an excellent observation you've made, even if it doesn't always hold.
    $endgroup$
    – John Omielan
    1 hour ago








  • 1




    $begingroup$
    I checked your result up to $n=533$ (for $n geq 534$, I have problems. Would you be interested by a huge table of $I(k)$ (I was able to generate it up to $k=540$). This is a very interesting problem.
    $endgroup$
    – Claude Leibovici
    59 mins ago












  • $begingroup$
    @ClaudeLeibovici: Thanks for checking, Claude! However, that table would be too huge for MSE. :)
    $endgroup$
    – Tito Piezas III
    56 mins ago














9












9








9


4



$begingroup$


This post discusses the integral,
$$I(k)=int_0^kpi(x)pi(k-x)dx$$



where $pi(x)$ is the prime-counting function. For example,
$$I(13)=int_0^{13}pi(x)pi(13-x)dx = 73$$



Using WolframAlpha, the first 50 values for $k=1,2,3,dots$ are,



$$I(k) = 0, 0, 0, 0, 1, 4, 8, 14, 22, 32, 45, 58, 73, 90, 110, 132, 158, 184, 214, 246, 282, 320, 363, 406, 455, 506, 562, 618, 678, 738, 804, 872, 944, 1018, 1099, 1180, 1269, 1358, 1450, 1544, 1644, 1744, 1852, 1962, 2078, 2196, 2321, 2446, 2581, 2718,dots$$



While trying to find if the above sequence obeyed a pattern, I noticed a rather unexpected relationship:






Q: For all $n>0$, is it true,
$$I(6n+4) - 2,I(6n+5) + I(6n+6) overset{color{red}?}= 0$$




Example, for $n=1,2$, then
$$I(10)-2I(11)+I(12)=32-2*45+58 = 0$$
$$I(16)-2I(17)+I(18)=132-2*158+184= 0$$
and so on.










share|cite|improve this question











$endgroup$




This post discusses the integral,
$$I(k)=int_0^kpi(x)pi(k-x)dx$$



where $pi(x)$ is the prime-counting function. For example,
$$I(13)=int_0^{13}pi(x)pi(13-x)dx = 73$$



Using WolframAlpha, the first 50 values for $k=1,2,3,dots$ are,



$$I(k) = 0, 0, 0, 0, 1, 4, 8, 14, 22, 32, 45, 58, 73, 90, 110, 132, 158, 184, 214, 246, 282, 320, 363, 406, 455, 506, 562, 618, 678, 738, 804, 872, 944, 1018, 1099, 1180, 1269, 1358, 1450, 1544, 1644, 1744, 1852, 1962, 2078, 2196, 2321, 2446, 2581, 2718,dots$$



While trying to find if the above sequence obeyed a pattern, I noticed a rather unexpected relationship:






Q: For all $n>0$, is it true,
$$I(6n+4) - 2,I(6n+5) + I(6n+6) overset{color{red}?}= 0$$




Example, for $n=1,2$, then
$$I(10)-2I(11)+I(12)=32-2*45+58 = 0$$
$$I(16)-2I(17)+I(18)=132-2*158+184= 0$$
and so on.







integration definite-integrals prime-numbers






share|cite|improve this question















share|cite|improve this question













share|cite|improve this question




share|cite|improve this question








edited 43 mins ago







Tito Piezas III

















asked 2 hours ago









Tito Piezas IIITito Piezas III

27.4k366174




27.4k366174












  • $begingroup$
    Note your proposed equation doesn't hold for $n = 0$ as $I(4) = 0$, $I(5) = 1$ and $I(6) = 4$.
    $endgroup$
    – John Omielan
    2 hours ago










  • $begingroup$
    @JohnOmielan: A typo. I meant all $n>0$. I will correct it.
    $endgroup$
    – Tito Piezas III
    1 hour ago










  • $begingroup$
    I have checked to confirm what you're asking is true for $n$ up to $18$. However, I have my doubts it'll always work, partially because it doesn't work for $n = 0$. Also, a similar type condition is that $I(6n) - 2I(6n + 1) + I(6n + 2) = 2$, which holds for $1 le n le 5$, but at $n = 6$, the LHS becomes $0$ instead. If I get a chance, I will investigate your equation to see if I can figure out why it's true for at least the first $18$ values and, more importantly, will it always stay true. Regardless, though, it's an excellent observation you've made, even if it doesn't always hold.
    $endgroup$
    – John Omielan
    1 hour ago








  • 1




    $begingroup$
    I checked your result up to $n=533$ (for $n geq 534$, I have problems. Would you be interested by a huge table of $I(k)$ (I was able to generate it up to $k=540$). This is a very interesting problem.
    $endgroup$
    – Claude Leibovici
    59 mins ago












  • $begingroup$
    @ClaudeLeibovici: Thanks for checking, Claude! However, that table would be too huge for MSE. :)
    $endgroup$
    – Tito Piezas III
    56 mins ago


















  • $begingroup$
    Note your proposed equation doesn't hold for $n = 0$ as $I(4) = 0$, $I(5) = 1$ and $I(6) = 4$.
    $endgroup$
    – John Omielan
    2 hours ago










  • $begingroup$
    @JohnOmielan: A typo. I meant all $n>0$. I will correct it.
    $endgroup$
    – Tito Piezas III
    1 hour ago










  • $begingroup$
    I have checked to confirm what you're asking is true for $n$ up to $18$. However, I have my doubts it'll always work, partially because it doesn't work for $n = 0$. Also, a similar type condition is that $I(6n) - 2I(6n + 1) + I(6n + 2) = 2$, which holds for $1 le n le 5$, but at $n = 6$, the LHS becomes $0$ instead. If I get a chance, I will investigate your equation to see if I can figure out why it's true for at least the first $18$ values and, more importantly, will it always stay true. Regardless, though, it's an excellent observation you've made, even if it doesn't always hold.
    $endgroup$
    – John Omielan
    1 hour ago








  • 1




    $begingroup$
    I checked your result up to $n=533$ (for $n geq 534$, I have problems. Would you be interested by a huge table of $I(k)$ (I was able to generate it up to $k=540$). This is a very interesting problem.
    $endgroup$
    – Claude Leibovici
    59 mins ago












  • $begingroup$
    @ClaudeLeibovici: Thanks for checking, Claude! However, that table would be too huge for MSE. :)
    $endgroup$
    – Tito Piezas III
    56 mins ago
















$begingroup$
Note your proposed equation doesn't hold for $n = 0$ as $I(4) = 0$, $I(5) = 1$ and $I(6) = 4$.
$endgroup$
– John Omielan
2 hours ago




$begingroup$
Note your proposed equation doesn't hold for $n = 0$ as $I(4) = 0$, $I(5) = 1$ and $I(6) = 4$.
$endgroup$
– John Omielan
2 hours ago












$begingroup$
@JohnOmielan: A typo. I meant all $n>0$. I will correct it.
$endgroup$
– Tito Piezas III
1 hour ago




$begingroup$
@JohnOmielan: A typo. I meant all $n>0$. I will correct it.
$endgroup$
– Tito Piezas III
1 hour ago












$begingroup$
I have checked to confirm what you're asking is true for $n$ up to $18$. However, I have my doubts it'll always work, partially because it doesn't work for $n = 0$. Also, a similar type condition is that $I(6n) - 2I(6n + 1) + I(6n + 2) = 2$, which holds for $1 le n le 5$, but at $n = 6$, the LHS becomes $0$ instead. If I get a chance, I will investigate your equation to see if I can figure out why it's true for at least the first $18$ values and, more importantly, will it always stay true. Regardless, though, it's an excellent observation you've made, even if it doesn't always hold.
$endgroup$
– John Omielan
1 hour ago






$begingroup$
I have checked to confirm what you're asking is true for $n$ up to $18$. However, I have my doubts it'll always work, partially because it doesn't work for $n = 0$. Also, a similar type condition is that $I(6n) - 2I(6n + 1) + I(6n + 2) = 2$, which holds for $1 le n le 5$, but at $n = 6$, the LHS becomes $0$ instead. If I get a chance, I will investigate your equation to see if I can figure out why it's true for at least the first $18$ values and, more importantly, will it always stay true. Regardless, though, it's an excellent observation you've made, even if it doesn't always hold.
$endgroup$
– John Omielan
1 hour ago






1




1




$begingroup$
I checked your result up to $n=533$ (for $n geq 534$, I have problems. Would you be interested by a huge table of $I(k)$ (I was able to generate it up to $k=540$). This is a very interesting problem.
$endgroup$
– Claude Leibovici
59 mins ago






$begingroup$
I checked your result up to $n=533$ (for $n geq 534$, I have problems. Would you be interested by a huge table of $I(k)$ (I was able to generate it up to $k=540$). This is a very interesting problem.
$endgroup$
– Claude Leibovici
59 mins ago














$begingroup$
@ClaudeLeibovici: Thanks for checking, Claude! However, that table would be too huge for MSE. :)
$endgroup$
– Tito Piezas III
56 mins ago




$begingroup$
@ClaudeLeibovici: Thanks for checking, Claude! However, that table would be too huge for MSE. :)
$endgroup$
– Tito Piezas III
56 mins ago










1 Answer
1






active

oldest

votes


















10












$begingroup$

The answer is yes. Sketch of solution:
$$
I(k) = int_0^k sum_{ple x} sum_{qle k-x} 1 ,dx = sum_p sum_{qle k-p} int_p^{k-q} dx = sum_p sum_{qle k-p} (k-(p+q)) = sum_{mle k} r(m)(k-m),
$$

where $r(m)$ is the number of ways of writing $m$ as the sum of two primes. Then
$$
I(6n+6)-2I(6n+5)+I(6n+4) = sum_{mle 6n+4} r(m)big( (6n+6-m)-2(6n+5-m)+(6m+4-m) big) + r(6n+5) = \0 + r(6n+5);
$$

and $r(6n+5)=0$ for every $nge1$, since the only way the odd integer $6n+5$ can be the sum of two primes is $6n+5=2+(6n+3)$, but $6n+3=3(2n+1)$ is always composite when $nge1$.



The same argument gives $I(6n+2)-2I(6n+1)+I(6n) = r(6n+1)$, which is $2$ if $6n-1$ is prime and $0$ otherwise; this is why (as observed by John Omielan) it equals $2$ for $1le nle 5$ but $0$ for $n=6$.






share|cite|improve this answer











$endgroup$









  • 1




    $begingroup$
    MSE never ceases to amaze me how fast some people can figure out the answer.
    $endgroup$
    – Tito Piezas III
    44 mins ago










  • $begingroup$
    Greg, do you know how to address Ultradark's question regarding when $I(k)$ is prime?
    $endgroup$
    – Tito Piezas III
    35 mins ago










  • $begingroup$
    This really surprises me since I thought the equation will be eventually false...
    $endgroup$
    – Seewoo Lee
    29 mins ago











Your Answer





StackExchange.ifUsing("editor", function () {
return StackExchange.using("mathjaxEditing", function () {
StackExchange.MarkdownEditor.creationCallbacks.add(function (editor, postfix) {
StackExchange.mathjaxEditing.prepareWmdForMathJax(editor, postfix, [["$", "$"], ["\\(","\\)"]]);
});
});
}, "mathjax-editing");

StackExchange.ready(function() {
var channelOptions = {
tags: "".split(" "),
id: "69"
};
initTagRenderer("".split(" "), "".split(" "), channelOptions);

StackExchange.using("externalEditor", function() {
// Have to fire editor after snippets, if snippets enabled
if (StackExchange.settings.snippets.snippetsEnabled) {
StackExchange.using("snippets", function() {
createEditor();
});
}
else {
createEditor();
}
});

function createEditor() {
StackExchange.prepareEditor({
heartbeatType: 'answer',
autoActivateHeartbeat: false,
convertImagesToLinks: true,
noModals: true,
showLowRepImageUploadWarning: true,
reputationToPostImages: 10,
bindNavPrevention: true,
postfix: "",
imageUploader: {
brandingHtml: "Powered by u003ca class="icon-imgur-white" href="https://imgur.com/"u003eu003c/au003e",
contentPolicyHtml: "User contributions licensed under u003ca href="https://creativecommons.org/licenses/by-sa/3.0/"u003ecc by-sa 3.0 with attribution requiredu003c/au003e u003ca href="https://stackoverflow.com/legal/content-policy"u003e(content policy)u003c/au003e",
allowUrls: true
},
noCode: true, onDemand: true,
discardSelector: ".discard-answer"
,immediatelyShowMarkdownHelp:true
});


}
});














draft saved

draft discarded


















StackExchange.ready(
function () {
StackExchange.openid.initPostLogin('.new-post-login', 'https%3a%2f%2fmath.stackexchange.com%2fquestions%2f3128367%2fa-curious-equality-of-integrals-involving-the-prime-counting-function%23new-answer', 'question_page');
}
);

Post as a guest















Required, but never shown

























1 Answer
1






active

oldest

votes








1 Answer
1






active

oldest

votes









active

oldest

votes






active

oldest

votes









10












$begingroup$

The answer is yes. Sketch of solution:
$$
I(k) = int_0^k sum_{ple x} sum_{qle k-x} 1 ,dx = sum_p sum_{qle k-p} int_p^{k-q} dx = sum_p sum_{qle k-p} (k-(p+q)) = sum_{mle k} r(m)(k-m),
$$

where $r(m)$ is the number of ways of writing $m$ as the sum of two primes. Then
$$
I(6n+6)-2I(6n+5)+I(6n+4) = sum_{mle 6n+4} r(m)big( (6n+6-m)-2(6n+5-m)+(6m+4-m) big) + r(6n+5) = \0 + r(6n+5);
$$

and $r(6n+5)=0$ for every $nge1$, since the only way the odd integer $6n+5$ can be the sum of two primes is $6n+5=2+(6n+3)$, but $6n+3=3(2n+1)$ is always composite when $nge1$.



The same argument gives $I(6n+2)-2I(6n+1)+I(6n) = r(6n+1)$, which is $2$ if $6n-1$ is prime and $0$ otherwise; this is why (as observed by John Omielan) it equals $2$ for $1le nle 5$ but $0$ for $n=6$.






share|cite|improve this answer











$endgroup$









  • 1




    $begingroup$
    MSE never ceases to amaze me how fast some people can figure out the answer.
    $endgroup$
    – Tito Piezas III
    44 mins ago










  • $begingroup$
    Greg, do you know how to address Ultradark's question regarding when $I(k)$ is prime?
    $endgroup$
    – Tito Piezas III
    35 mins ago










  • $begingroup$
    This really surprises me since I thought the equation will be eventually false...
    $endgroup$
    – Seewoo Lee
    29 mins ago
















10












$begingroup$

The answer is yes. Sketch of solution:
$$
I(k) = int_0^k sum_{ple x} sum_{qle k-x} 1 ,dx = sum_p sum_{qle k-p} int_p^{k-q} dx = sum_p sum_{qle k-p} (k-(p+q)) = sum_{mle k} r(m)(k-m),
$$

where $r(m)$ is the number of ways of writing $m$ as the sum of two primes. Then
$$
I(6n+6)-2I(6n+5)+I(6n+4) = sum_{mle 6n+4} r(m)big( (6n+6-m)-2(6n+5-m)+(6m+4-m) big) + r(6n+5) = \0 + r(6n+5);
$$

and $r(6n+5)=0$ for every $nge1$, since the only way the odd integer $6n+5$ can be the sum of two primes is $6n+5=2+(6n+3)$, but $6n+3=3(2n+1)$ is always composite when $nge1$.



The same argument gives $I(6n+2)-2I(6n+1)+I(6n) = r(6n+1)$, which is $2$ if $6n-1$ is prime and $0$ otherwise; this is why (as observed by John Omielan) it equals $2$ for $1le nle 5$ but $0$ for $n=6$.






share|cite|improve this answer











$endgroup$









  • 1




    $begingroup$
    MSE never ceases to amaze me how fast some people can figure out the answer.
    $endgroup$
    – Tito Piezas III
    44 mins ago










  • $begingroup$
    Greg, do you know how to address Ultradark's question regarding when $I(k)$ is prime?
    $endgroup$
    – Tito Piezas III
    35 mins ago










  • $begingroup$
    This really surprises me since I thought the equation will be eventually false...
    $endgroup$
    – Seewoo Lee
    29 mins ago














10












10








10





$begingroup$

The answer is yes. Sketch of solution:
$$
I(k) = int_0^k sum_{ple x} sum_{qle k-x} 1 ,dx = sum_p sum_{qle k-p} int_p^{k-q} dx = sum_p sum_{qle k-p} (k-(p+q)) = sum_{mle k} r(m)(k-m),
$$

where $r(m)$ is the number of ways of writing $m$ as the sum of two primes. Then
$$
I(6n+6)-2I(6n+5)+I(6n+4) = sum_{mle 6n+4} r(m)big( (6n+6-m)-2(6n+5-m)+(6m+4-m) big) + r(6n+5) = \0 + r(6n+5);
$$

and $r(6n+5)=0$ for every $nge1$, since the only way the odd integer $6n+5$ can be the sum of two primes is $6n+5=2+(6n+3)$, but $6n+3=3(2n+1)$ is always composite when $nge1$.



The same argument gives $I(6n+2)-2I(6n+1)+I(6n) = r(6n+1)$, which is $2$ if $6n-1$ is prime and $0$ otherwise; this is why (as observed by John Omielan) it equals $2$ for $1le nle 5$ but $0$ for $n=6$.






share|cite|improve this answer











$endgroup$



The answer is yes. Sketch of solution:
$$
I(k) = int_0^k sum_{ple x} sum_{qle k-x} 1 ,dx = sum_p sum_{qle k-p} int_p^{k-q} dx = sum_p sum_{qle k-p} (k-(p+q)) = sum_{mle k} r(m)(k-m),
$$

where $r(m)$ is the number of ways of writing $m$ as the sum of two primes. Then
$$
I(6n+6)-2I(6n+5)+I(6n+4) = sum_{mle 6n+4} r(m)big( (6n+6-m)-2(6n+5-m)+(6m+4-m) big) + r(6n+5) = \0 + r(6n+5);
$$

and $r(6n+5)=0$ for every $nge1$, since the only way the odd integer $6n+5$ can be the sum of two primes is $6n+5=2+(6n+3)$, but $6n+3=3(2n+1)$ is always composite when $nge1$.



The same argument gives $I(6n+2)-2I(6n+1)+I(6n) = r(6n+1)$, which is $2$ if $6n-1$ is prime and $0$ otherwise; this is why (as observed by John Omielan) it equals $2$ for $1le nle 5$ but $0$ for $n=6$.







share|cite|improve this answer














share|cite|improve this answer



share|cite|improve this answer








edited 45 mins ago









Tito Piezas III

27.4k366174




27.4k366174










answered 58 mins ago









Greg MartinGreg Martin

35.5k23364




35.5k23364








  • 1




    $begingroup$
    MSE never ceases to amaze me how fast some people can figure out the answer.
    $endgroup$
    – Tito Piezas III
    44 mins ago










  • $begingroup$
    Greg, do you know how to address Ultradark's question regarding when $I(k)$ is prime?
    $endgroup$
    – Tito Piezas III
    35 mins ago










  • $begingroup$
    This really surprises me since I thought the equation will be eventually false...
    $endgroup$
    – Seewoo Lee
    29 mins ago














  • 1




    $begingroup$
    MSE never ceases to amaze me how fast some people can figure out the answer.
    $endgroup$
    – Tito Piezas III
    44 mins ago










  • $begingroup$
    Greg, do you know how to address Ultradark's question regarding when $I(k)$ is prime?
    $endgroup$
    – Tito Piezas III
    35 mins ago










  • $begingroup$
    This really surprises me since I thought the equation will be eventually false...
    $endgroup$
    – Seewoo Lee
    29 mins ago








1




1




$begingroup$
MSE never ceases to amaze me how fast some people can figure out the answer.
$endgroup$
– Tito Piezas III
44 mins ago




$begingroup$
MSE never ceases to amaze me how fast some people can figure out the answer.
$endgroup$
– Tito Piezas III
44 mins ago












$begingroup$
Greg, do you know how to address Ultradark's question regarding when $I(k)$ is prime?
$endgroup$
– Tito Piezas III
35 mins ago




$begingroup$
Greg, do you know how to address Ultradark's question regarding when $I(k)$ is prime?
$endgroup$
– Tito Piezas III
35 mins ago












$begingroup$
This really surprises me since I thought the equation will be eventually false...
$endgroup$
– Seewoo Lee
29 mins ago




$begingroup$
This really surprises me since I thought the equation will be eventually false...
$endgroup$
– Seewoo Lee
29 mins ago


















draft saved

draft discarded




















































Thanks for contributing an answer to Mathematics Stack Exchange!


  • Please be sure to answer the question. Provide details and share your research!

But avoid



  • Asking for help, clarification, or responding to other answers.

  • Making statements based on opinion; back them up with references or personal experience.


Use MathJax to format equations. MathJax reference.


To learn more, see our tips on writing great answers.




draft saved


draft discarded














StackExchange.ready(
function () {
StackExchange.openid.initPostLogin('.new-post-login', 'https%3a%2f%2fmath.stackexchange.com%2fquestions%2f3128367%2fa-curious-equality-of-integrals-involving-the-prime-counting-function%23new-answer', 'question_page');
}
);

Post as a guest















Required, but never shown





















































Required, but never shown














Required, but never shown












Required, but never shown







Required, but never shown

































Required, but never shown














Required, but never shown












Required, but never shown







Required, but never shown







Popular posts from this blog

Щит и меч (фильм) Содержание Названия серий | Сюжет |...

is 'sed' thread safeWhat should someone know about using Python scripts in the shell?Nexenta bash script uses...

Meter-Bus Содержание Параметры шины | Стандартизация |...